LSAT and Law School Admissions Forum

Get expert LSAT preparation and law school admissions advice from PowerScore Test Preparation.

User avatar
 Dave Killoran
PowerScore Staff
  • PowerScore Staff
  • Posts: 5852
  • Joined: Mar 25, 2011
|
#94649
Setup and Rule Diagram Explanation

This is a Basic Linear: Balanced game.

The initial scenario establishes that seven toy-truck models will be assembled on seven numbered lines, creating a Basic Linear setup:

PT30-Dec 1999 LGE-G4_srd1.png

The first rule establishes that F is assembled on a lower-numbered line than J:

PT30-Dec 1999 LGE-G4_srd2.png

Remember, the “greater than” sign we use does not refer to numerical value; it refers to which variable is to the left of the other. This rule creates two Not Laws:

PT30-Dec 1999 LGE-G4_srd3.png

The second rule creates a block:

PT30-Dec 1999 LGE-G4_srd4.png

The block also produces two Not Laws:

PT30-Dec 1999 LGE-G4_srd5.png

The third rule creates a split-option for H, and the fourth rule assigns S to line 4, resulting in the final diagram (with K notated as a random):

PT30-Dec 1999 LGE-G4_srd6.png

One of the keys to this game is to realize that the MG block is limited in placement, and that when it is before S then M or G must be in 2, and when it is after S then M or G must be in 6. You could create four basic templates to reflect the four positions of the MG block (1-2, 2-3, 5-6, 6-7), but the game appears so simple this is probably unnecessary.
You do not have the required permissions to view the files attached to this post.
 c-erv
  • Posts: 15
  • Joined: Oct 14, 2016
|
#30179
Hello,

I found this game easy to set up and do, however I was not able to make any deductions / inferences after setting up the game and writing down the rules. I just want to make sure I didn't miss any key deduction from the rules that would of made the questions faster to speed through.

Thanks!
User avatar
 Dave Killoran
PowerScore Staff
  • PowerScore Staff
  • Posts: 5852
  • Joined: Mar 25, 2011
|
#30181
Hey c-erv,

Thanks for the question! This game is actually as easy as it feels. There are not laws for J and G on 1, and for F and M on 7, and then one for M on 3 and one for G on 5 (due to the interactions of the MG block with S in 4) but aside from that, not much else!

One of the keys to this game is to realize that the MG block is limited in placement, and that when it is before S then M or G must be in 2, and when it is after S then M or G must be in 6. You could create four basic templates to reflect the four positions of the MG block (1-2, 2-3, 5-6, 6-7), but the game appears so simple this is probably unnecessary.

Please let me know if that helps. Thanks!
User avatar
 Dania_ha
  • Posts: 8
  • Joined: May 25, 2021
|
#87554
Hello,

I'm having trouble understanding the second rule in this game:

"M is assembled on the line numbered one lower than the line on which G is assembled."

Now I know that the numbers go from highest ---> lowest, so the way I diagrammed the above rule is as so: (GM), as M is one number lower than that of G (i.e. G is higher). So I am confused as to why the actual diagram should be (MG), doesn't this make M on a higher numbered line than G?

I appreciate the help :)

Dania
 Robert Carroll
PowerScore Staff
  • PowerScore Staff
  • Posts: 1787
  • Joined: Dec 06, 2013
|
#87564
Dania,

The diagramming of the block as MG actually depends on the assumption that the diagram should have the lowest number on the left side, the reverse of what you posted. It's acceptable to diagram the base either way, but the order of the block is determined by which direction the base is in. Most people would want to diagram #1 on the far left side, which we at PowerScore also recommend, and that means the block should indeed be in the order "MG".

Robert Carroll

Get the most out of your LSAT Prep Plus subscription.

Analyze and track your performance with our Testing and Analytics Package.